Saltar al contenido principal
LibreTexts Español

1.6: Ecuaciones Diofantinas

  • Page ID
    113169
  • \( \newcommand{\vecs}[1]{\overset { \scriptstyle \rightharpoonup} {\mathbf{#1}} } \) \( \newcommand{\vecd}[1]{\overset{-\!-\!\rightharpoonup}{\vphantom{a}\smash {#1}}} \)\(\newcommand{\id}{\mathrm{id}}\) \( \newcommand{\Span}{\mathrm{span}}\) \( \newcommand{\kernel}{\mathrm{null}\,}\) \( \newcommand{\range}{\mathrm{range}\,}\) \( \newcommand{\RealPart}{\mathrm{Re}}\) \( \newcommand{\ImaginaryPart}{\mathrm{Im}}\) \( \newcommand{\Argument}{\mathrm{Arg}}\) \( \newcommand{\norm}[1]{\| #1 \|}\) \( \newcommand{\inner}[2]{\langle #1, #2 \rangle}\) \( \newcommand{\Span}{\mathrm{span}}\) \(\newcommand{\id}{\mathrm{id}}\) \( \newcommand{\Span}{\mathrm{span}}\) \( \newcommand{\kernel}{\mathrm{null}\,}\) \( \newcommand{\range}{\mathrm{range}\,}\) \( \newcommand{\RealPart}{\mathrm{Re}}\) \( \newcommand{\ImaginaryPart}{\mathrm{Im}}\) \( \newcommand{\Argument}{\mathrm{Arg}}\) \( \newcommand{\norm}[1]{\| #1 \|}\) \( \newcommand{\inner}[2]{\langle #1, #2 \rangle}\) \( \newcommand{\Span}{\mathrm{span}}\)\(\newcommand{\AA}{\unicode[.8,0]{x212B}}\)

    El volumen 2 de la Historia de la teoría de los números de Dickson trata de ecuaciones diofantinas. Es tan grande como los otros dos volúmenes combinados. Por lo tanto, es claro que no vamos a cubrir gran parte de este terreno en esta sección. Limitaremos nuestra atención a algunos problemas que son interesantes aunque no de importancia central.

    Uno de esos problemas es la ecuación Diofantina\(n! + 1 = x^2\) mencionada en una sección anterior. El problema se remonta a 1885 cuando H. Brochard conjeturó que las únicas soluciones son\(4!+1 = 52, 5!+1 = 112\) y\(7!+1 = 712\). Hacia 1895 Ramanujan hizo la misma conjetura pero no avanzó hacia una solución del problema. Hacia 1940 el problema apareció como un elemental (!!) problema en el Mensual. No se ofrecieron soluciones. No obstante en 1950 se publicó una solución incorrecta y desde entonces se han hecho varios intentos fallidos para probar el resultado. Nuevamente, alrededor de 1950 alguien se tomó la molestia de comprobar, por fuerza bruta, la conjetura hasta\(n = 50\). No obstante, antes, en su libro sobre la teoría de los números Kraitchik ya había probado el resultado hasta 5000. Por lo que sabemos ahí es donde se encuentran los problemas. Ahora daremos una indicación del método de Kraitchik.

    Supongamos que queremos verificar\(100! + 1\). Trabajando (mod 103) tenemos

    \(100! (-2)(-2) \equiv -1,\)\(100! + \dfrac{1}{2} \equiv 0\),\(100! + 1 \equiv \dfrac{1}{2} \equiv 52.\)

    Si ahora 52 es un no residuo de 103 hemos logrado nuestro objetivo. De lo contrario podríamos llevar a cabo un cálculo similar con otro\(p > 100\), digamos 107. Tenga en cuenta que\(100! + 1 \equiv 0\) (mod 101) no da información. Variaciones de este método se pueden utilizar para eliminar muchos números al por mayor y esto es lo que hizo Kraitchik. Ahora esbozamos una prueba que solo\(n! + 1 = x^8\) tiene un número finito de soluciones. Esta prueba depende de dos hechos que no hemos probado:

    1) Todo divisor de primos impares de\(x^2 + 1\) es de la forma\(4n + 1\);

    (2) Hay aproximadamente tantos primos\(4n + 1\) como\(4n + 3\).

    Ahora\(n! + 1 = x^8\) da\(n! = x^8 - 1= (x^4 + 1) (x^2 + 1) (x^2 - 1)\); a la derecha el aporte de primos\(4k + 1\) y\(4k − 1\) es aproximadamente el mismo mientras que a la izquierda todos los factores primos impares de\((x^4 + 1) (x^2 + 1)\) i.e., sobre\((n!)^{3/4}\) del producto, son de la forma\(4n + 1\).

    Pasamos ahora a un problema bastante diferente. Tiene la ecuación

    \(1^n + 2^n + \cdot\cdot\cdot + (m - 1)^n = m^n\)

    cualquier solución en enteros distintos de 1 + 2 = 3? Aquí hay algunas soluciones cercanas:

    \(3^2 + 4^2 = 5^2\),

    \(3^3 + 4^3 + 5^3 = 6^3\),

    \(1^6 + 2^6 + 4^6 + 7^6 + 9^6 + 12^6 + 13^6 + 15^6 + 16^6 + 18^6 + 20^6 + 22^6 + 23^6 = 28^6\).

    Ahora esbozamos una prueba de que si existen otras soluciones entonces\(m > 10^{1000000}\). El resto de esta sección apareció originalmente como el artículo “Sobre la ecuación diofantina”\(1^n + 2^n + \cdot\cdot\cdot + (m - 1)^n = m^n\), Scripta Mathematica, 19 (1953), pp. 84—88. (Pieter Moree discute este teorema y prueba en “Un sombrero de copa para los cuatro conejos matemáticos de Moser”, The American Mathematical Monthly, 118 (2011), 364— 370.)

    Desde hace tiempo se conocen varias ecuaciones aisladas que expresan la suma de\(n^{\text{th}}\) las potencias de los números enteros como una\(n^{\text{th}}\) potencia de un entero. Algunos ejemplos son:

    \(3^3 + 4^3 + 5^3 = 6^3\)

    \(\sum_{i = 1}^{100} i^4 - 1^4 - 2^4 - 3^4 - 8^4 - 10^4 - 72^4 = 212^4\)

    \(1^6 + 2^6 + 4^6 + 5^6 + 6^6 + 7^6 + 9^6 + 12^6 + 13^6 + 15^6 + 16^6 + 18^6 + 20^6 + 21^6 + 22^6 + 23^6 = 28^6\)

    Otros ejemplos y referencias a dichos resultados se dan en [1, p. 682]. Por otro lado la única solución conocida en números enteros a la ecuación en el título es la trivial 1 + 2 = 3. En una carta dirigida al autor, P. Erd\(\ddot{o}\) s conjeturó que esta es la única solución. El objeto de esta nota ts para mostrar que si la ecuación tiene una solución con\(n > 1\), entonces\(m > 10^{1000000}\).

    Vamos a\(S_n(m)\) denotar\(\sum_{i = 1}^{m - 1} i^n\). En lo que sigue suponemos

    \[S_n(m) \equiv m^n, n > 1.\]

    Es posible examinar (6.1) con diversos módulos y así obtener restricciones sobre\(m\) y\(n\). Este es esencialmente nuestro método, pero los módulos son tan elegidos que somos capaces de combinar las congruencias resultantes para obtener límites extremadamente grandes para\(m\) sin computación laboriosa.

    Utilizamos el siguiente lema.

    Lema 1. Si\(p\) es un primo y\(\epsilon_n(p)\) se define por\(\epsilon_n (p) = -1\) cuándo\((p - 1)\ |\ n\) y\(\epsilon_n (p) = 0\) cuándo\((p - 1)\) no se divide\(n\) entonces

    \[S_n(p) \equiv \epsilon_n(p)\ \ \ \ (\text{mod } p).\]

    Una prueba simple de (2) se da en [2, p. 90].

    Ahora supongamos\(p\ |\ (m - 1)\), entonces

    \(s_n(m) = \sum_{i = 0}^{\dfrac{m - 1}{p} - 1} \sum_{j = 1}^{p} (j + ip)^n \equiv \dfrac{m - 1}{p} \cdot \epsilon_n (p)\)(mod\(p\)).

    Por otro lado\(m \equiv 1\) (mod\(p\)) para que por (6.1)

    \[\dfrac{m - 1}{p} \cdot \epsilon_n (p) \equiv 1\ \ \ \ (\text{mod } p).\]

    De ahí\(\epsilon(p) \not\equiv 0\) (mod\(p\)) para que de la definición de\(\epsilon_n (p)\) ello se deduce que\(\epsilon_n(p) = -1\) y

    \[p\ |\ (m - 1)\ \ \text{implies } (p - 1)\ |\ n.\]

    Así (6.3) se puede poner en la forma

    \[\dfrac{m - 1}{p} + 1 \equiv 0\ \ \ \ (\text{mod } p)\]

    o

    \[m - 1 + p \equiv 0 \ \ \ \ (\text{mod } p^2).\]

    De (6.6) se deduce que\(m - 1\) es cuadrarefree. Además, ya que se comprueba fácilmente que se\(m - 1 \ne 2\) deduce que\(m - 1\) debe tener al menos un divisor primo, por lo que por (6.4)\(n\) es par.

    Ahora multiplicamos todas las congruencias del tipo (6.5), es decir, una por cada división primo\(m − 1\). Dado que\(m − 1\) es libre de cuadrados, el módulo resultante es\(m − 1\). Además, los productos que contengan dos o más factores distintos de la forma\((m − 1)/p\) serán divisibles por\(m − 1\). Así obtenemos

    \[(m - 1) \sum_{p\ |\ (m - 1)} \dfrac{1}{p} + 1 \equiv 0\ \ \ (\text{mod } m - 1)\]

    o

    \[\sum_{p\ |\ (m - 1)} \dfrac{1}{p} + \dfrac{1}{m - 1} \equiv 0\ \ \ (\text{mod } 1).\]

    Los únicos valores de los\(m \le 1000\) cuales satisfacen (6.8) son 3, 7, 43.

    Se procede a desarrollar tres congruencias más, similares a (6.8), que al combinarse con (6.8) conducen al resultado principal. La ecuación (6.1) se puede escribir en la forma

    \[S_n (m + 2) = 2m^n + (m + 1)^n.\]

    Supongamos que\(p\ |\ (m + 1)\). Usando (6.2) y el facto que\(n\) es parejo, obtenemos como antes

    \[p\ |\ (m + 1)\text{ implies } (p - 1)\ |\ n\]

    y

    \[\dfrac{m + 1}{p} + 2 \equiv 0 \ \ \ \ (\text{mod } p).\]

    De (6.11) se deduce que no aparece primo impar con el exponente mayor de una pulg\(m + 1\). El prime 2 sin embargo, requiere una atención especial. Si examinamos (1) con módulo 4, y usamos el hecho de que n es par, entonces encontramos eso\(m + 1 \equiv 1\) o 4 (mod 8). Así\(m + 1\) es impar o contiene 2 exactamente a la segunda potencia. Si asumimos la segunda de estas posibilidades entonces (6.11) se puede poner en la forma

    \[\dfrac{m + 1}{2p} + 1 \equiv 0\ \ \ \ (\text{mod }p).\]

    Multiplicamos todas las congruencias del tipo (12). Este módulo se convierte entonces\(\dfrac{m + 1}{2}\). Además, cualquier término que involucre dos o más factores distintos\(\dfrac{m + 1}{2p}\) será divisible por\(\dfrac{m + 1}{2}\) para que en la simplificación obtengamos

    \[\sum_{p\ |\ (m + 1)} \dfrac{1}{p} + \dfrac{2}{m + 1} \equiv 0\ \ \ \ (\text{mod } 1).\]

    Se procede a encontrar dos o más congruencias similares a (6.13) sin utilizar la suposición que\(m + 1\) es par. Supongamos eso\(p\ |\ 2m - 1\) y vamos\(t = \dfrac{1}{2} (\dfrac{2m-1}{p} - 1)\). Claramente\(t\) es un número entero y\(m - 1 = tp + \dfrac{p - 1}{2}\). Ya que\(n\) es incluso\(a^n = (-a)^n\) así que

    \(S_n (\dfrac{p - 1}{2}) \equiv \dfrac{\epsilon_n (p)}{2}\)(mod\(p\)).

    Ahora

    \[S_n (m) = \sum_{i = 0}^{t - 1} \sum_{j = 1}^{p - 1} (j + ip)^n + \sum_{i = 1}^{(p - 1)/2} i^n \equiv (t + \dfrac{1}{2}) \epsilon_n (p) \text{ (mod } p).\]

    Por otro lado\(m^n \equiv 0\) (mod\(p\)) para que (6.1) y (6.14) impliquen\(\epsilon_n (p) \not\equiv 0\). De ahí\(p - 1/n\) y por el teorema de Fermat\(m^n \equiv 1\) (mod\(p\)). Así (6.1) y (6.14) rinden\(-(t + \dfrac{1}{2}) \equiv 1\) (mod\(p\)). Sustituyendo\(t\) por su valor y simplificando obtenemos

    \[\dfrac{2m - 1}{p} + 2 \equiv 0\ \ \ \ (\text{mod } p).\]

    Dado que\(2m - 1\) es impar (6.15) implica que\(2m - 1\) es cuadrarefree. Multiplicando congruencias de tipo (6.15), una por cada uno de los divisores\(r\) primos de\(2m - 1\) rendimientos

    \(2^{r - 1} ((2m - 1) \sum_{p\ |\ (2m - 1)} \dfrac{1}{p} + 2) \equiv 0\)(mod\(2m - 1\)).

    Dado que el módulo es impar, esto da

    \[\sum_{p\ |\ (2m - 1)} \dfrac{1}{p} + \dfrac{2}{2m - 1} \equiv 0 \ \ \ \ (\text{mod } 1).\]

    Finalmente se obtiene una congruencia correspondiente para la división de primos\(2m + 1\). Para ello escribimos (6.1) en el formulario

    \[S_n(m+1) = 2m^n.\]

    Supongamos\(p\ |\ 2m + 1\). Set\(v = \dfrac{1}{2} (\dfrac{2m + 1}{p} - 1)\). Utilizando de nuevo un argumento similar al empleado para obtener (6.16) nos encontramos con que\((p - 1)\ |\ n\) y\(2m + 1\) es cuadrarefree. Finalmente obatin

    \[\sum_{p\ |\ (2m + 1)} \dfrac{1}{p} + \dfrac{4}{2m + 1} \equiv 0\ \ \ \ (\text{mod } p).\]

    Asumimos de nuevo que\(m + 1\) es incluso así que (6.13) sostiene. Si ahora sumamos los lados izquierdos de (6.8), (6.13), (6.16) y (6.18) obtenemos un entero, al menos 4. Ningún primo\(p > 3\) puede dividir más de uno de los números\(m - 1\),\(m + 1\),\(2m - 1\),\(2m + 1\). Además, 2 y 3 pueden dividir st la mayoría de dos de estos números. De ahí que si\(M = (m - 1) (m + 1)(2m - 1) (2m + 1)\) entonces

    \[\sum_{p\ |\ M} \dfrac{1}{p} + \dfrac{1}{m - 1} + \dfrac{2}{m + 1} + \dfrac{2}{2m - 1} + \dfrac{4}{2m + 1} \ge 4 - \dfrac{1}{2} - \dfrac{1}{3}.\]

    Ya hemos visto que las únicas posibilidades para\(m\) con\(m \le 1000\) son 3, 7, y 43. Estos son fácilmente descartados por (6.16). Así (6.19) rinde

    \[\sum_{p\ |\ M} \dfrac{1}{p} > 3.16.\]

    De (6.20) se deduce que si\(\sum_{p \le x} \dfrac{1}{p} < 3.16\) entonces\(M > \prod_{p \le x} p\). Demostraremos el siguiente lema.

    lema 2

    \(\sum_{p \le 10^7} \dfrac{1}{p} < 3.16.\)

    Prueba

    Por cómputo directo

    \[\sum_{p \le 10^8} \dfrac{1}{p} < 2.18.\]

    De la tabla de Lehmer [3] y estimaciones explícitas para\(\pi (x)\) debido a Rosser [4] se puede verificar fácilmente que para\(10^3 < x < 10^7\)

    \[\pi(x) < \dfrac{1.2x}{\log x}.\]

    Ahora en [2, p. 339] se muestra que

    \[\sum_{p \le x} \dfrac{1}{p} = \dfrac{\pi (x)}{x} + \int_2^{\pi} \dfrac{\pi (x)}{x} dx.\]

    combinando (6.21), (6.22) y (6.23) da el resultado requerido.

    En [4] se demuestra que

    \[sum_{p \le x} \log p > (1 - \dfrac{1}{\log x}) x, x < e^{100}.\]

    De ahí

    \(\log M > \log \prod_{p \le 10^7} p = \sum_{p \le 10^7} \log p > (1 - \dfrac{1}{7 \log 10}) 10^7 > (.93) 10^7.\)

    Ahora\(M < 4n^2\) para que

    \(\log m > (\dfrac{\log M - \log 4}{2}) > (.231)10^7\)

    y\(m > e^{(.231) 10^7} > 10^{1000000}\).

    Volviendo al caso\(m - 1\) impar, observamos que en este no podemos usar (6.13). Dejando que\(N = (m - 1) (2m - 1) (2m + 1)\) obtengamos de (6.8), (6.16) y (6.18)

    \[\sum_{p\ |\ N} \dfrac{1}{p} + \dfrac{1}{m - 1} + \dfrac{2}{2m - 1} + \dfrac{4}{2m + 1} > 3 - \dfrac{1}{3}.\]

    Sin embargo, dado que el primo 2 no aparece en el lado izquierdo (6.25) es en realidad una condición más fuerte en m que lo es (6.19) así que en cualquier caso\(m > 10^{1000000}\).

    Referencias

    [1] L.E. Dickson, Historia de la teoría de los números, vol. 2

    [2] G. H. Hardy y E. M. Wright, Una introducción a la teoría de los números.

    [3] D.H. Lehmer, “Lista de números primos del 1 al 10,006,721.”

    [4] B. Rosser, “Límite explícito para algunas funciones de números primos”, Amer. Jorno. de Matemáticas. , 63 (1941), 211—232.


    This page titled 1.6: Ecuaciones Diofantinas is shared under a CC BY 4.0 license and was authored, remixed, and/or curated by Leo Moser (The Trilla Group) via source content that was edited to the style and standards of the LibreTexts platform; a detailed edit history is available upon request.